Formes quadratiques et expressions polynomiales

Durée : 8 mn

Note maximale : 10

Question

Déterminer si les applications \(q_{i}~(1\le i\le3)\) de \(\mathbb{R}^{2}\) dans \(\mathbb{R}\) suivantes sont des formes quadratiques :

  1. \(\forall u = (x,y)\in\mathbb{R}^{2}, q_{1}(u) = -x^{2}\)

  2. \(\forall u = (x,y)\in\mathbb{R}^{2}, q_{2}(u) = x^{2} + y^{2}\)

  3. \(\forall u = (x,y)\in\mathbb{R}^{2}, \displaystyle{ q_{3}(u) = \left\{\begin{array}{l l} -x^2  & \textrm{si }  y= 0 \\ x^{2} + y^{2}& \textrm{si }  y \ne 0  \end{array}\right.}\)

Solution

Une application \(q\) de \(\mathbb{R}^{2}\) dans \(\mathbb{R}\) est une forme quadratique sur \(\mathbb{R}^{2}\) si, pour tout vecteur \(u = (x,y)\) de \(\mathbb{R}^{2}\), \(q(u)\) est une expression polynomiale homogène de degré 2 par rapport aux coordonnées \(x\) et \(y\) de \(u\).

1. et 2. [3 points] Il est immédiat que \(q_{1}\) et \(q_{2}\) sont des formes quadratiques.

3. [7 points] On montre que \(q_{3}\) n'est pas une forme quadratique par une démonstration par l'absurde.

En effet, supposons que \(q_{3}\) soit une expression polynomiale homogène de degré 2 par rapport aux coordonnées d'un vecteur de \(\mathbb{R}^{2}\); alors il existerait des réels \(a\), \(b\) et \(c\) tels que :

\(\forall u = (x,y) \in \mathbb{R}^{2},\) \(q_{3}(u) = a x^{2} + b y^{2} + c x y.\)

Or les images des quatre vecteurs \(u_{1} = (1,0),\) \(u_{2} = (0,1),\) \(u_{3} = (1,1)\) et \(u_{4} = (1,2)\) de \(\mathbb{R}^{2}\) sont : \(q_{3}(u_{1}) = -1,\) \(q_{3}(u_{2}) = 1,\) \(q_{3}(u_{3}) = 2\) et \(q_{3}(u_{4}) = 5.\)

En utilisant l'expression \(\forall u = (x,y) \in \mathbb{R}^{2},\) \(q_{3}(u) = ax^{2} + by^{2} + cxy\), on obtient : \(q_{3}(u_{1}) = a\), \(q_{3}(u_{2}) = b\), \(q_{3}(u_{3}) = a + b + c\), \(q_{3}(u_{4}) = a + 4b + 2c\),

d'où le système à résoudre :

\(\begin{array}{ccc}\left\{\begin{array}{l l } a = -1  \\ b = 1 \\ a+b+c = 2 \\ a + 4b +2c = 5   \end{array}\right.&\Leftrightarrow&\left\{\begin{array}{l l } a = -1  \\ b = 1 \\ c = 2 \\ a + 4b +2c = 5   \end{array}\right.\end{array}\)

Ce système n'a pas de solution car \((-1) +4 \times 1 + 2 \times 2 = 7.\)

Donc l'application \(q_{3}\) n'est pas une forme quadratique.

Remarque

pour tout scalaire \(\lambda\) de \(\mathbb{R}\) et tout vecteur \(w\) de \(\mathbb{R}^{2}\) on a quand même l'égalité \(q_{3}(\lambda w) = \lambda^{2}q_{3}(w).\) La condition \(q(\lambda u) = \lambda^{2} q(u)\) est une condition nécessaire pour qu'une application \(q\) de \(E\) dans \(\mathbb{K}\) soit une forme quadratique mais ce n'est pas une condition suffisante.